show that $(1^p)^k+(2^p)^k+cdots+((p-1)^p)^k$ is divisible by $p$












0












$begingroup$


This will be reduced to $1^k+2^k+3^k+cdots+(p-1)^k$ by Fermat Theorem. I know that the sum of the reduced residue system of $n$ is divisible by $n$. What I need to show, though, is that $1^k+2^k+3^k+cdots+(p-1)^k$ is a reduced residue system of $p$. How to do so?



Is there any other methods to prove so?



Added



I thought of pairing elements such as $a$ and $p-a equiv -a$ to cancelled out. But this works for odd $k$ only.










share|cite|improve this question











$endgroup$








  • 2




    $begingroup$
    This is not clear. Are you claiming that, say, $2^{p^k}equiv 2^kpmod p$? But this is false...$2^{3^2}=2^9equiv 2not equiv 2^2pmod 3$.
    $endgroup$
    – lulu
    Dec 20 '18 at 16:54






  • 4




    $begingroup$
    No. Keep in mind that $a^{p^k}$ denotes $a^{(p^k)}$ and not $left( a^pright)^k$ (which, after all, would just be $a^{pk}$).
    $endgroup$
    – lulu
    Dec 20 '18 at 17:01






  • 2




    $begingroup$
    This problem is actually easier if you don't reduce using Fermat's theorem. You were on the right path with pairing, and for all primes greater than 2 your exponents will be odd. The case of 2 is a trivial case though.
    $endgroup$
    – mascoj
    Dec 20 '18 at 17:01






  • 1




    $begingroup$
    It is not a reduced residue system for $p=7$ and $k=3$.
    $endgroup$
    – lhf
    Dec 20 '18 at 17:07






  • 6




    $begingroup$
    Possible duplicate of $ 1^k+2^k+3^k+...+(p-1)^k $ always a multiple of $p$?
    $endgroup$
    – lulu
    Dec 20 '18 at 17:17
















0












$begingroup$


This will be reduced to $1^k+2^k+3^k+cdots+(p-1)^k$ by Fermat Theorem. I know that the sum of the reduced residue system of $n$ is divisible by $n$. What I need to show, though, is that $1^k+2^k+3^k+cdots+(p-1)^k$ is a reduced residue system of $p$. How to do so?



Is there any other methods to prove so?



Added



I thought of pairing elements such as $a$ and $p-a equiv -a$ to cancelled out. But this works for odd $k$ only.










share|cite|improve this question











$endgroup$








  • 2




    $begingroup$
    This is not clear. Are you claiming that, say, $2^{p^k}equiv 2^kpmod p$? But this is false...$2^{3^2}=2^9equiv 2not equiv 2^2pmod 3$.
    $endgroup$
    – lulu
    Dec 20 '18 at 16:54






  • 4




    $begingroup$
    No. Keep in mind that $a^{p^k}$ denotes $a^{(p^k)}$ and not $left( a^pright)^k$ (which, after all, would just be $a^{pk}$).
    $endgroup$
    – lulu
    Dec 20 '18 at 17:01






  • 2




    $begingroup$
    This problem is actually easier if you don't reduce using Fermat's theorem. You were on the right path with pairing, and for all primes greater than 2 your exponents will be odd. The case of 2 is a trivial case though.
    $endgroup$
    – mascoj
    Dec 20 '18 at 17:01






  • 1




    $begingroup$
    It is not a reduced residue system for $p=7$ and $k=3$.
    $endgroup$
    – lhf
    Dec 20 '18 at 17:07






  • 6




    $begingroup$
    Possible duplicate of $ 1^k+2^k+3^k+...+(p-1)^k $ always a multiple of $p$?
    $endgroup$
    – lulu
    Dec 20 '18 at 17:17














0












0








0





$begingroup$


This will be reduced to $1^k+2^k+3^k+cdots+(p-1)^k$ by Fermat Theorem. I know that the sum of the reduced residue system of $n$ is divisible by $n$. What I need to show, though, is that $1^k+2^k+3^k+cdots+(p-1)^k$ is a reduced residue system of $p$. How to do so?



Is there any other methods to prove so?



Added



I thought of pairing elements such as $a$ and $p-a equiv -a$ to cancelled out. But this works for odd $k$ only.










share|cite|improve this question











$endgroup$




This will be reduced to $1^k+2^k+3^k+cdots+(p-1)^k$ by Fermat Theorem. I know that the sum of the reduced residue system of $n$ is divisible by $n$. What I need to show, though, is that $1^k+2^k+3^k+cdots+(p-1)^k$ is a reduced residue system of $p$. How to do so?



Is there any other methods to prove so?



Added



I thought of pairing elements such as $a$ and $p-a equiv -a$ to cancelled out. But this works for odd $k$ only.







number-theory elementary-number-theory modular-arithmetic divisibility






share|cite|improve this question















share|cite|improve this question













share|cite|improve this question




share|cite|improve this question








edited Dec 20 '18 at 17:02







Maged Saeed

















asked Dec 20 '18 at 16:48









Maged SaeedMaged Saeed

8771417




8771417








  • 2




    $begingroup$
    This is not clear. Are you claiming that, say, $2^{p^k}equiv 2^kpmod p$? But this is false...$2^{3^2}=2^9equiv 2not equiv 2^2pmod 3$.
    $endgroup$
    – lulu
    Dec 20 '18 at 16:54






  • 4




    $begingroup$
    No. Keep in mind that $a^{p^k}$ denotes $a^{(p^k)}$ and not $left( a^pright)^k$ (which, after all, would just be $a^{pk}$).
    $endgroup$
    – lulu
    Dec 20 '18 at 17:01






  • 2




    $begingroup$
    This problem is actually easier if you don't reduce using Fermat's theorem. You were on the right path with pairing, and for all primes greater than 2 your exponents will be odd. The case of 2 is a trivial case though.
    $endgroup$
    – mascoj
    Dec 20 '18 at 17:01






  • 1




    $begingroup$
    It is not a reduced residue system for $p=7$ and $k=3$.
    $endgroup$
    – lhf
    Dec 20 '18 at 17:07






  • 6




    $begingroup$
    Possible duplicate of $ 1^k+2^k+3^k+...+(p-1)^k $ always a multiple of $p$?
    $endgroup$
    – lulu
    Dec 20 '18 at 17:17














  • 2




    $begingroup$
    This is not clear. Are you claiming that, say, $2^{p^k}equiv 2^kpmod p$? But this is false...$2^{3^2}=2^9equiv 2not equiv 2^2pmod 3$.
    $endgroup$
    – lulu
    Dec 20 '18 at 16:54






  • 4




    $begingroup$
    No. Keep in mind that $a^{p^k}$ denotes $a^{(p^k)}$ and not $left( a^pright)^k$ (which, after all, would just be $a^{pk}$).
    $endgroup$
    – lulu
    Dec 20 '18 at 17:01






  • 2




    $begingroup$
    This problem is actually easier if you don't reduce using Fermat's theorem. You were on the right path with pairing, and for all primes greater than 2 your exponents will be odd. The case of 2 is a trivial case though.
    $endgroup$
    – mascoj
    Dec 20 '18 at 17:01






  • 1




    $begingroup$
    It is not a reduced residue system for $p=7$ and $k=3$.
    $endgroup$
    – lhf
    Dec 20 '18 at 17:07






  • 6




    $begingroup$
    Possible duplicate of $ 1^k+2^k+3^k+...+(p-1)^k $ always a multiple of $p$?
    $endgroup$
    – lulu
    Dec 20 '18 at 17:17








2




2




$begingroup$
This is not clear. Are you claiming that, say, $2^{p^k}equiv 2^kpmod p$? But this is false...$2^{3^2}=2^9equiv 2not equiv 2^2pmod 3$.
$endgroup$
– lulu
Dec 20 '18 at 16:54




$begingroup$
This is not clear. Are you claiming that, say, $2^{p^k}equiv 2^kpmod p$? But this is false...$2^{3^2}=2^9equiv 2not equiv 2^2pmod 3$.
$endgroup$
– lulu
Dec 20 '18 at 16:54




4




4




$begingroup$
No. Keep in mind that $a^{p^k}$ denotes $a^{(p^k)}$ and not $left( a^pright)^k$ (which, after all, would just be $a^{pk}$).
$endgroup$
– lulu
Dec 20 '18 at 17:01




$begingroup$
No. Keep in mind that $a^{p^k}$ denotes $a^{(p^k)}$ and not $left( a^pright)^k$ (which, after all, would just be $a^{pk}$).
$endgroup$
– lulu
Dec 20 '18 at 17:01




2




2




$begingroup$
This problem is actually easier if you don't reduce using Fermat's theorem. You were on the right path with pairing, and for all primes greater than 2 your exponents will be odd. The case of 2 is a trivial case though.
$endgroup$
– mascoj
Dec 20 '18 at 17:01




$begingroup$
This problem is actually easier if you don't reduce using Fermat's theorem. You were on the right path with pairing, and for all primes greater than 2 your exponents will be odd. The case of 2 is a trivial case though.
$endgroup$
– mascoj
Dec 20 '18 at 17:01




1




1




$begingroup$
It is not a reduced residue system for $p=7$ and $k=3$.
$endgroup$
– lhf
Dec 20 '18 at 17:07




$begingroup$
It is not a reduced residue system for $p=7$ and $k=3$.
$endgroup$
– lhf
Dec 20 '18 at 17:07




6




6




$begingroup$
Possible duplicate of $ 1^k+2^k+3^k+...+(p-1)^k $ always a multiple of $p$?
$endgroup$
– lulu
Dec 20 '18 at 17:17




$begingroup$
Possible duplicate of $ 1^k+2^k+3^k+...+(p-1)^k $ always a multiple of $p$?
$endgroup$
– lulu
Dec 20 '18 at 17:17










0






active

oldest

votes











Your Answer





StackExchange.ifUsing("editor", function () {
return StackExchange.using("mathjaxEditing", function () {
StackExchange.MarkdownEditor.creationCallbacks.add(function (editor, postfix) {
StackExchange.mathjaxEditing.prepareWmdForMathJax(editor, postfix, [["$", "$"], ["\\(","\\)"]]);
});
});
}, "mathjax-editing");

StackExchange.ready(function() {
var channelOptions = {
tags: "".split(" "),
id: "69"
};
initTagRenderer("".split(" "), "".split(" "), channelOptions);

StackExchange.using("externalEditor", function() {
// Have to fire editor after snippets, if snippets enabled
if (StackExchange.settings.snippets.snippetsEnabled) {
StackExchange.using("snippets", function() {
createEditor();
});
}
else {
createEditor();
}
});

function createEditor() {
StackExchange.prepareEditor({
heartbeatType: 'answer',
autoActivateHeartbeat: false,
convertImagesToLinks: true,
noModals: true,
showLowRepImageUploadWarning: true,
reputationToPostImages: 10,
bindNavPrevention: true,
postfix: "",
imageUploader: {
brandingHtml: "Powered by u003ca class="icon-imgur-white" href="https://imgur.com/"u003eu003c/au003e",
contentPolicyHtml: "User contributions licensed under u003ca href="https://creativecommons.org/licenses/by-sa/3.0/"u003ecc by-sa 3.0 with attribution requiredu003c/au003e u003ca href="https://stackoverflow.com/legal/content-policy"u003e(content policy)u003c/au003e",
allowUrls: true
},
noCode: true, onDemand: true,
discardSelector: ".discard-answer"
,immediatelyShowMarkdownHelp:true
});


}
});














draft saved

draft discarded


















StackExchange.ready(
function () {
StackExchange.openid.initPostLogin('.new-post-login', 'https%3a%2f%2fmath.stackexchange.com%2fquestions%2f3047737%2fshow-that-1pk2pk-cdotsp-1pk-is-divisible-by-p%23new-answer', 'question_page');
}
);

Post as a guest















Required, but never shown

























0






active

oldest

votes








0






active

oldest

votes









active

oldest

votes






active

oldest

votes
















draft saved

draft discarded




















































Thanks for contributing an answer to Mathematics Stack Exchange!


  • Please be sure to answer the question. Provide details and share your research!

But avoid



  • Asking for help, clarification, or responding to other answers.

  • Making statements based on opinion; back them up with references or personal experience.


Use MathJax to format equations. MathJax reference.


To learn more, see our tips on writing great answers.




draft saved


draft discarded














StackExchange.ready(
function () {
StackExchange.openid.initPostLogin('.new-post-login', 'https%3a%2f%2fmath.stackexchange.com%2fquestions%2f3047737%2fshow-that-1pk2pk-cdotsp-1pk-is-divisible-by-p%23new-answer', 'question_page');
}
);

Post as a guest















Required, but never shown





















































Required, but never shown














Required, but never shown












Required, but never shown







Required, but never shown

































Required, but never shown














Required, but never shown












Required, but never shown







Required, but never shown







Popular posts from this blog

Aardman Animations

Are they similar matrix

Ficheiro:Flag of Oman.svg